PT67.S4.Q4 - economist consumers benefit monopoly

Josh4737Josh4737 Free Trial Member
edited December 2015 in Logical Reasoning 4 karma
Hello, I was hoping you could help me out with this question. I originally had it down to C and E and chose E for the following reason:
Wouldn't answer choice E strengthen the conclusion in that companies that obtain would be more likely to perform the actions the economist says they will? More specifically, should E not be true, then would the argument possibly fall apart? The stimulus claims that companies CAN do this and that, which in turn benefits the consumer. Yes the companies can perform those actions but what if they don't? Doesn't answer choice E bridge the gap and make it more likely that companies will actually perform the actions they have the ability to do due to the monopoly?
http://7sage.com/lsat_explanations/lsat-67-section-4-question-04/

Comments

Sign In or Register to comment.